6
$\begingroup$

I am interested bounding the following quantity. Given fixed $k \in \mathbb{N}$, $a,b \in \mathbb{Z}$, $\sigma \in [0,1)$, and intervals $I_1,I_2 \subset \mathbb{Z}$ can we establish the bound

$$S = \sum_{x \in I_1,y \in I_2}\min\{(ax^k+by^k,q)^{1/k},q^{\sigma}\} \ll q^{\varepsilon}(|I_1||I_2|+q^{\sigma})?$$

The reason this bound seems like it might reasonable is because most of the time we have that $(ax^k,q) \neq (by^k,q)$, when this is case we have that $(ax^k+by^k,q)^{1/k} \ll \min\{(x,q),(y,q)\}$. We then get that $S \ll S_1+S_2$ where $$S_1 = \sum_{x \in I_1,y \in I_2}\min\{(x,q),(y,q),q^{\sigma}\} \ll q^{\varepsilon}(|I_1||I_2|+q^{\sigma}),$$ and $$S_2 = \sum_{\substack{x \in I_1,y \in I_2 \\ (ax^k,q) = (by^k,q)}}\min\{(ax^k+by^k,q)^{1/k},q^{\sigma}\}.$$ It feels like $S_2$ should be smaller than $S_1$ because of the GCD restriction, but I am having trouble seeing if this is actually the case or not.

One reason this might not be possible is if there is always a positive proportion of points $x,y$ in $I_1 \times I_2$ for which $(ax^k,q) = (by^k,q)$ and $(ax^k+by^k,q)^{1/k} \gg q^{\alpha}$ for some $\alpha > 0$, then we get the lower bound

$$S_2 \gg q^{\min\{\sigma,\alpha\}}|I_1||I_2|.$$

I am not sure if this is the case or not, I am not very familiar with these types of sums.

$\endgroup$

2 Answers 2

3
+100
$\begingroup$

If $q$ is prime, then put $U_1 = \{(x,y) \in I_1 \times I_2 : q \vert ax^k + by^k\}$ and $U_2 = I_1 \times I_2 \setminus U_1$. Then $U_1$ is of the form $I_1 \times I_2 \cap L$, where $L$ is a union of at most $k$ lattices $\Lambda_\omega$ of the form

$$\displaystyle \Lambda_\omega = \{(x,y) \in \mathbb{Z}^2 : x \equiv \omega y \pmod{q}\}$$

where $a\omega^k + b \equiv 0 \pmod{q}$. We have

$$\displaystyle |I_1 \times I_2 \cap \Lambda_\omega| = \frac{|I_1||I_2|}{q} + O(\max\{|I_1|, |I_2|\}).$$

The contribution of the sum coming from $U_2$ is clearly $O(|I_1||I_2|)$. On the other hand,

$$\displaystyle \sum_{(x,y) \in U_1} \min\{\gcd(ax^k + by^k, q)^{1/k}, q^\sigma\} = \sum_{(x,y) \in U_1} \min\{q^{1/k}, q^\sigma\}$$ $$ \leq k \min\{q^{1/k}, q^\sigma\} \left(\frac{|I_1||I_2|}{q} + O(\max\{|I_1|, |I_2\})\right).$$

In general, if one of the lattices $\Lambda_{\omega}$ has a very short vector, which is the case say when $q = a + b$, then the big-$O$ term is sharp. Therefore in general I don't think this bound can be improved to completely remove the dependence on $\max\{|I_1|, |I_2|\}$, but probably in most interesting regimes this is harmless.

If you drop the requirement that $q$ is prime, then a bit more work has to be done to sort the sum by divisors of $q$, but basically the same type of bound can be obtained.

$\endgroup$
3
  • $\begingroup$ A similar argument seems to work for squarefree q, I am unsure how the this lattice argument can be extended to q which is not squarefree since the reduction to linear congruences does not seem to be as simple. $\endgroup$ Commented Oct 3, 2024 at 21:12
  • $\begingroup$ @DanielFlores For non-square free $q$ it suffices to note that you can use Hensel's lemma to lift roots modulo $p$ to roots modulo $p^k$ uniquely; in fact they must correspond to linear factors of $ax^k + by^k$ over $\mathbb{Q}_p$. This works always provided that a root mod $p$ exists and $p$ does not divide the discriminant of $ax^k + by^k$, which in this case is just a power of $ab$ times some constant factor depending only on $k$ I think. This should allow the argument to generalize to arbitrary $q$. $\endgroup$ Commented Oct 4, 2024 at 16:01
  • 1
    $\begingroup$ Yes, you're right! thank you so much for your help. $\endgroup$ Commented Oct 4, 2024 at 19:01
0
$\begingroup$

Let for $d \mid q$ take $$U_d(I_1,I_2) = \{x \in I_1,y \in I_2: ax^k+bx^k \equiv 0 \mod d\},$$ then one has that $$S \le \sum_{d |q}\min\{d^{1/k},q^{\sigma}\}\#U_d(I_1,I_2).$$ So the problem reduces to estimating $\#U_d(I_1,I_2)$, this may be done with exponential sums since \begin{align*} \#U_d(I_1,I_2) &= \frac{1}{d}\sum_{1 \le c \le d} \sum_{\substack{x \in I_1 \\ y \in I_2}}e\left(\frac{c}{d}(ax^k+by^k) \right) \\ &= \frac{1}{d}\sum_{1 \le c \le d} \sum_{1 \le x,y \le d}e\left(\frac{c}{d}(ax^k+by^k) \right)\left(\frac{|I_1||I_2|}{d^2}+ O \left(\frac{|I_1|+|I_2|}{d} \right) \right) \\ &\ll d^{-2/k}\left(|I_1||I_2|+ d(|I_1|+|I_2|) \right). \end{align*} Where the last bound comes from the well known bound $$\sum_{1 \le x \le d}e\left(\frac{c}{d}ax^k \right) \ll (a,d)^{1/k}d^{1-1/k} \ll d^{1-1/k}.$$ Thus we obtain the bound \begin{align*} S &\ll \sum_{d |q}\min\{d^{1/k},q^{\sigma}\}d^{-2/k}\left(|I_1||I_2|+ d(|I_1|+|I_2|) \right) \\ &\le |I_1||I_2|\sum_{d|q}d^{-1/k}+q^{\sigma+1-2/k}(|I_1|+|I_2|)\sum_{d \mid q}1 \\ &\ll q^{\epsilon}\left( |I_1||I_2|+ q^{\sigma+1-2/k}(|I_1|+|I_2|)\right). \end{align*} This bound is worse than the one Stanley hints at for $k>2$, however I have not been able to completely figure out how his argument generalizes to general $q$. For now this bound is actually sufficient for me since I am most interested in the case $k=2$.

$\endgroup$

You must log in to answer this question.

Start asking to get answers

Find the answer to your question by asking.

Ask question

Explore related questions

See similar questions with these tags.